1975 IMO Problems/Problem 1: Difference between revisions
Bobwang001 (talk | contribs) Ph.D degree, https://www.youtube.com/@math000 |
No edit summary |
||
| Line 1: | Line 1: | ||
==Problem== | ==Problem== | ||
Let <math>x_i, y_i</math> <math>(i=1,2,\cdots,n)</math> be real numbers such that <cmath>x_1\ge x_2\ge\cdots\ge x_n \text{ and } y_1\ge y_2\ge\cdots\ge y_n.</cmath> Prove that, if <math>z_1, z_2,\cdots, z_n</math> is any permutation of <math>y_1, y_2, \cdots, y_n,</math> then <cmath>\sum^n_{i=1}(x_i-y_i)^2\le\sum^n_{i=1}(x_i-z_i)^2.</cmath> | Let <math>x_i, y_i</math> <math>(i=1,2,\cdots,n)</math> be real numbers such that <cmath>x_1\ge x_2\ge\cdots\ge x_n \text{ and } y_1\ge y_2\ge\cdots\ge y_n.</cmath> Prove that, if <math>z_1, z_2,\cdots, z_n</math> is any permutation of <math>y_1, y_2, \cdots, y_n,</math> then <cmath>\sum^n_{i=1}(x_i-y_i)^2\le\sum^n_{i=1}(x_i-z_i)^2.</cmath> | ||
==Video Solution(In Chinese)== | ==Video Solution(In Chinese)== | ||
https://youtu.be/3-iF4zIxsUQ | https://youtu.be/3-iF4zIxsUQ | ||
==Solution== | ==Solution== | ||
We can expand and simplify the inequality a bit, and using the fact that <math>z</math> is a permutation of <math>y</math>, we can cancel some terms. | We can expand and simplify the inequality a bit, and using the fact that <math>z</math> is a permutation of <math>y</math>, we can cancel some terms. | ||
<cmath>\sum^n_{i=1}x_i^2 + \sum^n_{i=1}y_i^2 - 2\sum^n_{i=1}x_iy_i \leq \sum^n_{i=1}x_i^2 + \sum^n_{i=1}z_i^2 - 2\sum^n_{i=1}x_iz_i</cmath> | <cmath>\sum^n_{i=1}x_i^2 + \sum^n_{i=1}y_i^2 - 2\sum^n_{i=1}x_iy_i \leq \sum^n_{i=1}x_i^2 + \sum^n_{i=1}z_i^2 - 2\sum^n_{i=1}x_iz_i</cmath> | ||
| Line 17: | Line 20: | ||
~mathboy100 | ~mathboy100 | ||
==Solution 2== | ==Solution 2== | ||
We can rewrite the summation as | We can rewrite the summation as | ||
<cmath>\sum^n_{i=1} x_i^2 + \sum^n_{i=1} y_i^2 - \sum^n_{i=1}x_iy_i \le \sum^n_{i=1} x_i^2 + \sum^n_{i=1} z_i^2 - \sum^n_{i=1}x_iz_i.</cmath> | <cmath>\sum^n_{i=1} x_i^2 + \sum^n_{i=1} y_i^2 - \sum^n_{i=1}x_iy_i \le \sum^n_{i=1} x_i^2 + \sum^n_{i=1} z_i^2 - \sum^n_{i=1}x_iz_i.</cmath> | ||
| Line 29: | Line 34: | ||
is proved, which is equivalent to what we wanted to prove. | is proved, which is equivalent to what we wanted to prove. | ||
<cmath> </cmath> | <cmath> </cmath> | ||
~Imajinary | ~Imajinary | ||
==Remark== | ==Remark== | ||
It is only the most common way of rearrangement inequality after expanding and subtracting same terms.~bluesoul | It is only the most common way of rearrangement inequality after expanding and subtracting same terms.~bluesoul | ||
==Remarks (added by pf02, November 2025)== | |||
<math>\mathbf{1}</math> The first solution can not be called a solution. | |||
There are several issues with it. The first issue is that the | |||
notions of "pairing" and "type 1 move" vs. "type 2 move" are | |||
not defined. This is particularly bad with "type 1" and | |||
"type 2" "moves" since the way they are used contradicts the | |||
fact that they are meant to be complementary. Indeed, we can | |||
not have only strict inequalities. | |||
The second issue is with the last paragraph. Consider the | |||
sentences "Suppose that <math>x_a</math> is the biggest <math>x</math>-value that | |||
is not paired with its <math>y</math>-value in the <math>x</math> and <math>z</math> pairing. | |||
Then, switch this <math>y</math>-value with the <math>y</math>-value currently | |||
paired with <math>x_a</math>." These two sentences do not make sense. | |||
Furthermore, depending on what meaning we would want to force | |||
on these two sentences, and on what "type 2 move" may mean, | |||
the sentence which follows, "This is obviously a type 2 move" | |||
may or may not be true. And finally, the sentence "Continue | |||
this process until you reach back to the <math>x</math>-<math>y</math> pairing" | |||
is unclear: it is not clear that we would reach "the <math>x</math>-<math>y</math> | |||
pairing" (whatever "pairing" may mean). | |||
<math>\mathbf{2}</math> Solution 2 is incorrect. The author assumed | |||
that <math>x_pz_m</math> and <math>x_qz_n</math> are terms in <math>\sum^n_{i=1}x_iz_i</math>, | |||
but this is not true unless <math>p = m</math> and <math>q = n</math>. | |||
(Never mind the fact that there is a crucial sentence which | |||
does not make sense: "assume, for contradiction, that neither | |||
of those conditions are true and that there exists some order | |||
of <math>z_i</math>s that are not ordered in the form | |||
<math>z_1 \ge z_2 \ge \ldots \ge z_n</math> such that | |||
<math>\sum^n_{i=1}x_iz_i</math> is at a maximum out of all possible | |||
permutations and is greater than the sum <math>\sum^n_{i=1}x_iy_i</math>". | |||
But a benign and eager reader can guess what the author most | |||
likely wanted to say.) | |||
<math>\mathbf{3}</math> The remark which follows the two "solutions" makes | |||
no sense at all. | |||
<math>\mathbf{4}</math> Below I will give a solution to the problem. | |||
==Solution 3== | |||
Let <math>S = \{y_1, y_2, \cdots, y_n\}</math> be the given set of numbers, | |||
<math>y_1 \ge y_2 \ge \cdots \ge y_n</math>, and let <math>\{z_1, z_2, \cdots, z_n\}</math> | |||
be a permutation of the set <math>S</math>. In other words <math>z_j = \sigma(y_i)</math> | |||
for a one-to-one function <math>\sigma: S \rightarrow S</math>. Remember that | |||
a transposition is a permutation such that there exist <math>i, j</math> such | |||
that <math>\sigma(y_i) = y_j, \sigma(y_j) = y_i</math> and <math>\sigma(y_k) = y_k</math> | |||
for all <math>k \ne i, j</math>. In other words, a transposition swaps two | |||
elements of <math>S</math>, and leaves the others in place. | |||
TO BE CONTINUED | |||
==See Also== | ==See Also== | ||
{{IMO box|year=1975|before=First Question|num-a=3}} | {{IMO box|year=1975|before=First Question|num-a=3}} | ||
[[Category:Olympiad | [[Category:Olympiad Algebra Problems]] | ||
Revision as of 16:29, 3 November 2025
Problem
Let
be real numbers such that
Prove that, if
is any permutation of
then
Video Solution(In Chinese)
Solution
We can expand and simplify the inequality a bit, and using the fact that
is a permutation of
, we can cancel some terms.
Consider the pairing
,
, ...
. By switching around some of the
values, we have obtained the pairing
,
, ...
. Suppose that we switch around two
-values,
and
, such that
. If
, call this a type 1 move. Otherwise, call this a type 2 move.
Type 2 moves only increase the sum of the products of the pairs. The sum is increased by
. This is equivalent to
, which is clearly nonnegative since
and
.
We will now consider switching from the
-
pairing back to the
-
pairing. We will prove that from any pairing of
and
values, you can use just type 2 moves to navigate back to the pairing of
and
values, which will complete the proof.
Suppose that
is the biggest
-value that is not paired with its
-value in the
and
pairing. Then, switch this
-value with the
-value currently paired with
. This is obviously a type 2 move. Continue this process until you reach back to the
-
pairing. All moves are type 2 moves, so the proof is complete.
~mathboy100
Solution 2
We can rewrite the summation as
Since
, the above inequality is equivalent to
We will now prove that the left-hand side of the inequality is the greatest sum reached out of all possible values of
. Obviously, if
or
, the inequality is true. Now, assume, for contradiction, that neither of those conditions are true and that there exists some order of
s that are not ordered in the form
such that
is at a maximum out of all possible permutations and is greater than the sum
. This necessarily means that in the sum
there exists two terms
and
such that
and
. Notice that
which means if we make the terms
and
instead of the original
and
, we can achieve a higher sum. However, this is impossible, since we assumed we had the highest sum. Thus, the inequality
is proved, which is equivalent to what we wanted to prove.
~Imajinary
Remark
It is only the most common way of rearrangement inequality after expanding and subtracting same terms.~bluesoul
Remarks (added by pf02, November 2025)
The first solution can not be called a solution.
There are several issues with it. The first issue is that the
notions of "pairing" and "type 1 move" vs. "type 2 move" are
not defined. This is particularly bad with "type 1" and
"type 2" "moves" since the way they are used contradicts the
fact that they are meant to be complementary. Indeed, we can
not have only strict inequalities.
The second issue is with the last paragraph. Consider the
sentences "Suppose that
is the biggest
-value that
is not paired with its
-value in the
and
pairing.
Then, switch this
-value with the
-value currently
paired with
." These two sentences do not make sense.
Furthermore, depending on what meaning we would want to force
on these two sentences, and on what "type 2 move" may mean,
the sentence which follows, "This is obviously a type 2 move"
may or may not be true. And finally, the sentence "Continue
this process until you reach back to the
-
pairing"
is unclear: it is not clear that we would reach "the
-
pairing" (whatever "pairing" may mean).
Solution 2 is incorrect. The author assumed
that
and
are terms in
,
but this is not true unless
and
.
(Never mind the fact that there is a crucial sentence which
does not make sense: "assume, for contradiction, that neither
of those conditions are true and that there exists some order
of
s that are not ordered in the form
such that
is at a maximum out of all possible
permutations and is greater than the sum
".
But a benign and eager reader can guess what the author most
likely wanted to say.)
The remark which follows the two "solutions" makes
no sense at all.
Below I will give a solution to the problem.
Solution 3
Let
be the given set of numbers,
, and let
be a permutation of the set
. In other words
for a one-to-one function
. Remember that
a transposition is a permutation such that there exist
such
that
and
for all
. In other words, a transposition swaps two
elements of
, and leaves the others in place.
TO BE CONTINUED
See Also
| 1975 IMO (Problems) • Resources | ||
| Preceded by First Question |
1 • 2 • 3 • 4 • 5 • 6 | Followed by Problem 3 |
| All IMO Problems and Solutions | ||